Gastrointestinal System/Adomen/Abdomen Wall Flashcards
Page 489 - Page 592
In a small town in Sydney, suddenly a number of people fell sick with bloody diarrhea, severe abdominal pain, and oliguria. In the Emergency Department, most of them are severely dehydrated and confused. The panel of doctors recommended emergency blood tests. Which one of the following, if present on test results, is diagnostic of the condition?
A. Thrombocytopenia.
B. Microangiopathic hemolytic anemia.
C. High creatinine levels.
D. Severe ADAMTS 13 deficiency.
E. High LDH levels.
B. Microangiopathic hemolytic anemia.
-
Classic Presentations:
- HUS: Renal failure (oliguria), abdominal pain following invasive diarrhea.
- TTP: Similar symptoms to HUS but with additional CNS symptoms like confusion.
-
Causes:
- HUS: Often follows a diarrheal illness caused by bacteria like enterohemorrhagic E. coli O157:H7 and Shigella dysenteriae, which produce Shiga toxins.
- TTP: Linked to a deficiency in the enzyme ADAMTS13, leading to large von Willebrand factor (VWF) multimers causing microthrombi.
-
Bacterial Toxins:
- Shiga toxin and Shiga-like toxin can:
- Trigger chemokine/cytokine secretion in colonic and renal cells.
- Activate monocytes and platelets.
- Cause secretion of large VWF multimers in glomerular endothelial cells.
- Shiga toxin and Shiga-like toxin can:
-
Clinical Differentiation:
- HUS: More severe renal involvement.
- TTP: More common CNS involvement.
- History of diarrheal illness often precedes HUS.
-
Treatment Differences:
- HUS in children: Supportive care, dialysis if needed.
- TTP: Plasma exchange.
- HUS in adults: Often treated with plasma exchange like TTP.
-
Diagnosis:
- No single test; diagnosed based on clinical presentation and presence of microangiopathic hemolytic anemia.
- Key diagnostic signs include:
- Anemia
- Elevated bilirubin and LDH
- Presence of schistocytes on a peripheral smear
-
Microangiopathic Hemolysis: Indicates either HUS or TTP.
-
Not Diagnostic Alone:
- Thrombocytopenia: Seen in HUS, TTP, and other conditions like ITP.
- Renal Function Tests (e.g., creatinine): High in many conditions including dehydration.
- ADAMTS13 Test: Time-consuming, more prognostic than diagnostic, not routinely ordered.
- LDH: Elevated in hemolysis, not specific for HUS/TTP.
-
Not Diagnostic Alone:
- HUS: Renal failure, abdominal pain, follows diarrhea, supportive care in children.
- TTP: CNS symptoms, plasma exchange treatment.
- Diagnosis: Based on clinical presentation, anemia, elevated bilirubin and LDH, and schistocytes in blood smear.
By focusing on these key points and understanding the differences and overlaps between HUS and TTP, it becomes easier to remember and apply this information clinically.
Renal failure reflected by oliguria and abdominal pain following invasive diarrhea is classic presentation of hemolytic uremic syndrome (HUS)
. Addition of CNS symptoms (confusion) to this constellation makes thrombotic thrombocytopenic purpura (TTP)
another possibility if it is caused by ischemia because of thrombi and not sodium derangement caused by severe dehydration. The exact etiology of HUS and TTP is not clear, but the role of Shiga toxin in HUS and ADAMTS13 (a metalloproteinase) in TTP have been implicated.
HUS, and to some extent TTP, commonly occur following a diarrheal illness with enterohemorrhagic Escherichia coli O157:H7 and Shigella dysenteriae serotype I. These bacteria, in addition to causing bloody diarrhea, are capable of secreting Shiga (Shigella) and Shiga-like toxin (E-coli). These toxins can bind to certain cell membrane receptors, which, depending on the cell type, can result in:
1. Chemokine or cytokine secretion (colonic and renal epithelial cells)
2. Cellular activation (monocytes and platelets)
3. Secretion of unusually large Von Willebrand multimers (glomerular endothelial cells)
Clinical differentiation of hemolytic-uremic syndrome (HUS) and TTP can be problematic; however, central nervous system involvement is more common in TTP, and more severe renal involvement in HUS.
In HUS, an antecedent history of diarrheal illness is often present. In fact, some authors suggest a clinical classification of HUS based on the presence or absence of diarrhea.
In children, the distinction between HUS and TTP may be more important because general supportive measures (with dialysis as needed) are the standard therapy for HUS, while TTP is treated with plasma exchange. In adults, however, HUS is also often treated with plasma exchange; therefore, differentiating between HUS and TTP is not as important as it is in children.
There is not a single diagnostic test for HUS and TTP. These are diagnosed based on clinical presentation and presence of microangiopathic hemolytic anemia presenting with:1. Anemia, elevated bilirubin and LDH (often significantly high)
2. Presence of schistocytes on peripheral smear
In this case, presence of micro-angiopathic hemolysis, is the most important finding that suggests either HUS or TTP. The distinction between the two, however, neither is possible with certainty, nor is necessary as the therapeutic approach is almost the same for both HUS and TTP in adults.
Option A: Thrombocytopenia is almost always a feature of HUS and TTP, as it is in other conditions such as ITP; therefore, it is not diagnostic.
Option C: Renal function tests, including creatinine are part of workup for suspected HUS or TTP, but not diagnostic because high creatinine levels are seen in TTP and HUS as well as dehydration and many other conditions.
Option D: ADAMTS13 is a metalloproteinase that cleaves Von Willebrand factor (VWF). Its deficiency results in circulating large multimers of VWF. Large molecules of VWF multimers by adhering circulating platelets together leads to microthrombi in the organs, ischemia, and end organ damage. Majority of patients (>90%) with acquired TTP have circulating antibodies against ADAMTS13 making them ADAMTS13 deficient; however severe ADAMTS13 deficiency is more common in sporadic forms rather than outbreaks. As the test is time-consuming and more prognostic rather than diagnostic, it is not routinely ordered. Furthermore, ADAMTS13 deficiency alone does not seem to cause TTP, and a contributing factor such as pregnancy, infection, drugs, etc. is required to trigger TTP.
Option E: Regardless of the etiology, elevated LDH is seen in hemolysis. LDH is neither sensitive, nor specific for HUS/TTP.
Thrombotic Thrombocytopenic Purpura (TTP)
Which one of the following is not correct regarding hemochromatosis?
A. The incidence of hemochromatosis in Australia is 1:200.
B. C282Y homozygous accounts for approximately 90% of hemochromatosis cases in Australia.
C: The majority of patients with one copy of C282Y and H63D mutation never develop hemochromatosis.
D. Ninety percent of those with homozygous C282Y will develop symptoms at some stage of their lives.
E. Carriers of only one copy of the mutated HFE gene will not be affected clinically.
D. Ninety percent of those with homozygous C282Y will develop symptoms at some stage of their lives.
Option A: Correct - The incidence of hereditary hemochromatosis (HH) in Australia is 1 in 200-300.
Option B: Correct - Appromixately 80-90% of HH cases in Australia have homozygous C282Y mutations of their HFE gene.
Option C: Correct - Most of patients with heterozygous C282Y/H63D mutations never develop clinical symptoms or will have only mild symptoms.
Option D: Incorrect - Only 28.4% of males and 1.2% of the females with C282Y mutation will develop clinically significant presentation of iron overload some time in their lives.
TOPIC REVIEW
Hemochromatosis has two forms. It is either due a genetic condition, namely hereditary hemochromatosis (HH), or secondary to conditions leading to iron overload in the body such as chronic hemolysis and multiple transfusions. Secondary iron overload is referred to as hemosidrerosis.
HH is an autosomal recessive genetic disease with variable penetrance and delayed age of onset (rare before the age of 30 years), in which the body iron content exceeds enormously beyond the normal limit (20-60g compared to normal amount of 4g) due to increased absorption through gastrointestinal tract. HH is the result of a mutation in HFE gene, which is located on the short arm of chromosome 6.
The two most common mutations of HFE gene are termed C282Y and H63D. Different possible combinations of these mutations and their risk for development of clinical HH is summarized in the following table. (See photo)
Homozygous C282Y is the most common form of HH (80-90%), followed by heterozygous C282Y/H3D. 28.4% of males and 1.2% of the females with C282Y mutation will develop clinically significant presentation of iron overload some time in their lives, but rarely before the age of 30 years.
Those with homozygous H63D are very unlikely to develop clinical disease. Heterozygosity of C282Y and H63D leads to HH with milder clinical forms.
Epidemiology
The prevalence of hemochromatosis in the Australian of Northern Europe background is 1 in 200-300 (250). Every 1 in 8 is a silent carrier of one mutated HFE gene.
Clinical disease is seen more in men than women. The age of onset for women is more advanced. The age of onset is rarely before 30 years.
Pathophysiology
HH results in deposition of iron in different body organs leading to a multi-organ/multisystem involvement and presentation.
Presentation
Hemochromatosis, through iron overload and deposition of iron in several organ systems, can present with the following features:
- Chronic hepatitis and cirrhosis (most common cause of mortality)
- Abdominal pain
- Arthralgia – often MCPs and large joints, due to chondrocalcinosis and peusogout. The pain is similar to the pain associated with osteoarthritis.
- Skin darkening (tanned skin) – deposition of iron in the skin.
- Small testes, infertility, impotence and decreased libido –small testes are due to hypopituitarism and/or liver disease. Iron deposition in gonads occurs, but is not the cause hypogonadism.
- Damage to the pancreas and diabetes mellitus (known as bronze diabetes).
- Restrictive cardiomyopathy and congestive heart failure (15% of patients). Other less common cardiac manifestations are supraventricular tachycardias, atrial fibrillation, atrial flutter and varying degrees of atrioventricular block.
- Panhypopituitarism - caused by iron deposition in the pituitary
- Hepatocellular carcinoma (hepatoma) – in 10% of cases with liver involvement.
- Osteoporosis (25% of cases) and osteopenia (41% of case)
- Sparse body hair especially pubic hair (62% of patients)
- Spoon nails (50% of cases)
Diagnosis
The transferrin saturation (ratio of serum iron to iron binding capacity) reflects increased absorption of iron, which is the underlying biological defect in HH. A fasting transferrin saturation >45% is the most sensitive test for detecting early iron overload, but not diagnostic of HH. Ferritin can be used to assess iron overload, but it is not as accurate because it is an acute phase reactant and may be elevated in response to several physiologic stresses, alcohol consumption, and liver disease. Serum ferritin is abnormal when it is >250 μg/L in pre-menopausal women and >300 μg/L in men and post-menopausal women.
If fasting transferrin saturation or serum ferritin is increased on more than one occasion, HH should be suspected, even if there are no clinical symptoms or abnormal LFTs. In this situation, the HFE gene test should be considered as the next diagnostic tool.
Although liver biopsy is the most accurate test to diagnose hemochromatosis, an MRI of the liver in conjunction with HFE gene testing for mutations are diagnostic enough to eliminate the need for liver biopsy.
NOTE - Iron studies may be normal in individuals with a genetic predisposition to HH, who have not developed iron overload. Up to 40% of homozygotes have normal iron studies, which may be due to overt (blood donation) or covert (gynecological or gastrointestinal) blood loss.
A 42-year-old woman comes to your clinic seeking advice on screening for hemochromatosis. Her 32-year-old brother has been recently diagnosed with hereditary hemochromatosis. She has two children, aged 9 and 18 years old. Which one of the following is the best action regarding screening for hemochromatosis and assessing the chance of her children developing the disease?
A. Screen her for hemochromatosis.
B. Screen both children for hemochromatosis.
C. Screen the 18-year-old child for hemochromatosis.
D. Screening is not needed at this stage.
E. Screen her and his husband.
A. Screen her for hemochromatosis.
The gene involved in hereditary hemochromatosis (HH) is the HFE gene. Mutations in the HFE gene can lead to problems with iron storage, causing hemochromatosis. The two main types of HFE gene mutations are C282Y and H63D.
- Heterozygote: Has one copy of the mutated HFE gene (either C282Y or H63D).
- Homozygote: Has two copies of the C282Y mutation. (Note: Two copies of H63D mutation do not cause clinical hemochromatosis.)
- Compound Heterozygote: Has one C282Y mutation and one H63D mutation. These individuals are usually asymptomatic or have mild symptoms.
- HH follows an autosomal recessive pattern, meaning both parents must be carriers for their children to be at risk.
- If both parents are carriers (heterozygous), there is a 25% chance their children will be affected by HH.
- The woman’s brother has HH, so both parents are at least carriers of the HFE mutation.
- She could be a heterozygote, homozygote, or compound heterozygote. Genetic testing will determine her status.
- First-degree and second-degree relatives of individuals with HH should be tested with iron studies and the HFE gene test.
- Testing is necessary to understand the risk and decide on further steps.
- If she does not have a faulty HFE gene, no further testing of her children is needed.
- If she is homozygous for the C282Y mutation, her children should be tested, starting with the father. If the father has no HFE mutations, the children are only carriers.
- Testing the mother (the patient) for the HFE gene mutations is the next best step in management. This will help understand the risk for her children and determine if further testing is necessary.
The gene involved in hereditary hemochromatosis (HH) is the HFE gene. Mutations in the HFE gene can lead to problems with iron storage, causing hemochromatosis. The two main types of HFE gene mutations are C282Y and H63D.
- Heterozygote: Has one copy of the mutated HFE gene (either C282Y or H63D).
- Homozygote: Has two copies of the C282Y mutation. (Note: Two copies of H63D mutation do not cause clinical hemochromatosis.)
- Compound Heterozygote: Has one C282Y mutation and one H63D mutation. These individuals are usually asymptomatic or have mild symptoms.
- HH follows an autosomal recessive pattern, meaning both parents must be carriers for their children to be at risk.
- If both parents are carriers (heterozygous), there is a 25% chance their children will be affected by HH.
- The woman’s brother has HH, so both parents are at least carriers of the HFE mutation.
- She could be a heterozygote, homozygote, or compound heterozygote. Genetic testing will determine her status.
- First-degree and second-degree relatives of individuals with HH should be tested with iron studies and the HFE gene test.
- Testing is necessary to understand the risk and decide on further steps.
- If she does not have a faulty HFE gene, no further testing of her children is needed.
- If she is homozygous for the C282Y mutation, her children should be tested, starting with the father. If the father has no HFE mutations, the children are only carriers.
- Testing the mother (the patient) for the HFE gene mutations is the next best step in management. This will help understand the risk for her children and determine if further testing is necessary.
The gene involved in hereditary hemochromatosis (HH) is called the HFE gene. Mutations in the HFE gene can lead to impaired regulation of iron storage and clinical manifestations of hemochromatosis. There are two types of mutation in HFE gene: C282Y and H63D.
Terminology
- Those with only one copy of the mutated HFE (either C282Y or H63D) gene are called heterozygote.
- Those with two copies of C282Y mutation are called ‘homozygote’. Since those with two copies of H63D mutation never develop clinical hemochromatosis, homozygote, refers to a person with both copies of HFE with C282Y mutation only.
- Those with one copy of HFE gene with C282Y mutation and the other with H63D mutation are called ‘compound heterozygote’. These individuals are often asymptomatic and if symptoms are present they are mild.
About 90% of people of Northern European ancestry with symptoms of HH have the C282Y mutation in both copies of their HFE gene (homozygote). Two percent are compound heterozygote (see above).
Since HH follows an autosomal recessive pattern of inheritance, there is often no family history, or affected family members may appear to be scattered in generations. If both parents heterozygous (carriers for a mutation in the HFE gene), there is a 25% chance for their children to be affected and genetically predisposed to HH.
This woman’s brother is diagnosed with hereditary hemochromatosis; meaning that both their parents have been at least carriers of HFE gene mutation. She can be a heterozygote, homozygote or compound heterozygote. Genetic testing will reveal that.
It is recommended that first-degree and second-degree relatives of individuals, who have HH or are homozygous for the C282Y gene mutation, are tested with iron studies and the HFE gene test. Based on the recommendation she needs testing. Further steps depend on her test results:
- If she does not have a faulty HFE gene, no further testing of the children would be indicated, because even if the father is homozygote, the children would only be carriers in the worst case scenario.
- If she is found to be homozygote, then the next step would be testing the father. If the father is found to have no HFE mutations the children are not at risk of HH and could only be carriers and not susceptible to HH.
Of the options, testing the mother for HH would be the next best step in management.
A 35-year-old woman presents to your practice with complaint of right upper quadrant discomfort for the past 3 months. She smokes 10 cigarettes a day and drink alcohol at weekends. On examination, she is otherwise healthy, with no palpable abdominal mass or tenderness. An abdominal CT scan is arranged and obtained, which is shown in the accompanying photograph. Which one of the following is the most likely diagnosis?
A. Liver abscess.
B. Simple hepatic cyst.
C. Hepatic hemangioma.
D. Hepatocellular carcinoma.
E. Hydatid cyst.
B. Simple hepatic cyst.
The homogenous hypoattenuating (darker than the surrounding liver parenchyma) oval-shaped lesion in the photograph is characteristic of a simple hepatic cyst. Simple hepatic cysts are common benign liver lesions and have no malignant potential. They can be diagnosed on ultrasound, CT, or MRI.
Simple hepatic cysts are one of the most common liver lesions, occurring in approximately 2-7% of the population. It is slightly more common in women. Hepatic cysts are typically discovered incidentally and are almost always asymptomatic, unless they are large enough to cause symptoms (such as in this patient).
Simple hepatic cysts may be isolated or multiple and may vary from a few millimeters to several centimeters in diameter. Simple hepatic cysts are benign developmental lesions that do not communicate with the biliary tree. They can occur anywhere in the liver, but there may be a greater predilection for the right lobe of the liver.
Certain diseases are associated with multiple hepatic cysts and include:
- Polycystic liver disease
- Autosomal dominant polycystic kidney disease (ADPKD) - hepatic cysts may be seen in ~40% of those with ADPKD
- Von-Hippel-Lindau disease
Findings on ultrasonography include:
- Round or ovoid anechoic lesion (may be lobulated)
- Well-marginated with a thin or imperceptible wall and a clearly defined back wall
- May show posterior acoustic enhancement if large enough
- A few septa may be possible, but no wall thickening is present
- A small amount of layering debris is possible
- No internal vascularity on color Doppler
On CT scan, a hepatic cysts is characterized by its homogenous hypoattenuation (water attenuation). The wall is usually imperceptible, and the cyst is not enhanced after intravenous administration of contrast material.
Option A: Liver abscess is associated with fever, leukocytosis and more pronounced symptoms. They are solid and hyperattenuated on CT scan.
Option C: hemangiomas have less homogenocity and well-demarcation compared to hepatic simple cysts.
Option D: Hepatocellular carcinoma (HCC) present with less demarcated hepatic lesions that are often hypoattenuated on CT and hypoechoic on ultrasonography. The radiologic findings are inconsistent with HCC as a possible diagnosis.
Option E: Hydatid cysts presents as a multiloculated cyst (daughter cysts within the main cyst)
Which one of the following is the most common cause of a hyperechoic mass on liver ultrasonography?
A. Hepatoma.
B. Simple cyst.
C. Hemangioma.
D. Echinococcal cyst.
E. Metastatic liver disease.
Hyperechoic: light gray on the ultrasound (air, fat, fluid)
C. Hemangioma.
Hemangiomas, benign proliferation of vascular tissue, are the most common cause of a hyperechoic liver mass on ultrasound.
Hepatic hemangiomas (also known as hepatic venous malformations) are benign non-neoplastic hypervascualr lesions. They are frequently diagnosed as an incidental finding on imaging in asymptomatic patients. It is very important to differentiate hemangiomas from hepatic neoplasms.
On ultrasound, they typically manifest as well-defined hyperechoic lesions; however, a small proportion (10%) are hypoechoic, which may be due to a background of hepatic steatosis, where liver parenchyma has increased echogenicity.
On CT scan, most hemangiomas are relatively well defined.
Features of typical lesions on three phasic CT scan include:
- Noncontrast: often hypoattenuating relative to liver parenchyma
- Arterial phase: typically show discontinuous, nodular, peripheral enhancement (small lesions may show uniform enhancement)
- Portal venous phase: progressive peripheral enhancement with more centripetal fill in
- Delayed phase further irregular fill in and therefore iso- or hyper-attenuating to liver parenchyma
Hepatic Hemangioma
A 28-year-old man presents with increasing dysphagia and odynophagia. Endoscopy reveals inflamed esophagus, which easily bleeds on contact. Several biopsies are taken showing eosinophilic infiltrations on histology. Which one of the following would be the most appropriate next step in management?
A. Proton pump inhibitors.
B. Swallowed fluticasone.
C. Oral prednisolone.
D. Albendazole.
E. Helicobacter pylori eradication.
A. Proton pump inhibitors.
Primary eosinophilic esophagitis (EoE) is an increasingly recognized medical condition characterized clinically by symptoms related to esophageal dysfunction, and histologically by eosinophilic inflammation in the esophagus.
EoE is hypothesized to be an atopic inflammatory disease caused by an abnormal immune response to antigenic stimulation, mostly foods. Normally, eosinophils are normal component of mucosal infiltrates in all-length of the gastrointestinal tract except the esophagus. Eosinophils in the esophageal mucosa are always pathologic.
Generally, the clinical symptoms of EoE are nonspecific, and the patients are in good physical condition resulting in a delayed diagnosis (years) in some cases.
The presenting symptoms vary depending on the age of the onset:
Children - children tend to present with nausea and vomiting, weight loss, anemia, and failure to thrive. In neonates and infants, refusal of food is the most common presenting symptom.
Adults - the characteristic symptoms in adults include dysphagia for solid foods, retrosternal pain and food impaction. Some patients also present with gastroesophageal reflux disease (GERD) symptoms unresponsive to medical anti-reflux therapy. A subset of patients have been recognized to have a typical clinical presentation of EoE in the absence of GERD who show a clinicopathologic response to PPIs. This condition is currently referred to as PPI-responsive EoE.
There is no Australian guidelines for diagnosis and management of EoE and current recommendation is based on the guidelines by the American College of Gastroenterology (ACG).
According to the ACG, diagnostic criteria for EoE include all of the following:
- Symptoms related to esophageal dysfunction.
- ≥15 eosinophils/hpf on esophageal biopsy
- Persistence of eosinophilia after a proton pump inhibitor (PPI) trial
- Secondary cause of esophageal eosinophilia excluded
This patient has symptoms related to esophageal dysfunction (dysphagia and odynophagia) and established eosinophilia on histological studies. In order for EoE to be the definite diagnosis, it is necessary that eosinophilia persists after an 8-week trial of a PPI as well and other causes of eosinophilia are excluced.
The rationale behind the trial of PPI is that GERD may mimic EoE, coexist with it, or contribute to it. Conversely, EoE may contribute to GERD; therefore, the diagnosis of EoE is generally made after the symptoms persist after an 8-week course of proton pump inhibitors (PPIs) as the best initial step in management. PPIs may benefit patients with EoE either by reducing acid production in patients with co-existent GERD, or by other unknown anti-inflammatory mechanisms.
The main three components of treatment in established EoE are (1) dietary advice and alteration, (2) pharmacotherapy, and (3) surgical intervention. For pharmacological intervention topical swallowed steroids (e.g., fluticasone, budesonide) are considered the main treatment options, once the diagnosis of EoE is established either after failed PPI therapy or normal pH studies.
For patients unresponsive to the above measure, oral (systemic) predniso(lo)ne maybe indicated.
TOPIC REVIEW
Causes of esophageal eosinophilia:
- Eosinophilic esophagitis
- GERD
- PPI-responsive eosinophilic esophagitis
- Achalasia
- Crohn’s disease
- Parasitic infections
- Drug hypersensitivity
- Connective tissue disease (e.g., scleroderma, dermatomyositis)
- Celiac disease
- Hypereosinophilic syndrome
Eosinophilc esophagitis
A 70-year-old man presents with difficulty in swallowing for the past 6 months and 4 kilogram weight loss in this period. He describes that the most difficult part of swallowing for him is when he tries to start getting the food down his mouth. He had been a smoker for most of his adult life but has quit 10 years ago. Which one of the following would be the most appropriate management option at this point?
A. Endoscopy.
B. Surgery.
C. Upper series barium study.
D. Helicobacter pylori testing.
E. Manometry.
C. Upper series barium study.
No matter what the clues point towards, every patient with dysphagia should undergo appropriate investigation. Just because of weight loss, the patient’s cannot be told to have esophageal cancer. Although the patient’s age is a red flag for dysphagia, the fact that it occurs at initiation of swallowing makes oropharyngeal dysphagia a more likely probability. On the other hand, every patient with dysphagia, regardless of the etiology, may have weight loss due to decreased calorie intake; nonetheless, a thorough and judicious assessment should be considered for every patient with dysphagia.
The best initial step in management of dysphagia depends on provisional diagnosis based on the history and clinical findings. When esophageal cancer is suspected, evaluation starts with upper endoscopy and biopsy
. With oropharyngeal and motility-related dysphagia, barium studies
should come first.
In this scenario, oropharyngeal dysphagia, probably caused by a retropharyngeal pouch (Zenker’s diverticulum), is the most likely diagnosing; therefore, barium swallow would be the best initial assessment tool. If a retropharyngeal pouch is diagnosed on barium studies, endoscopy should be avoided due to the significant risk of the scope perforating the pouch.
Option A: Endoscopy is the initial investigation when cancer is suspected based on history and clinical features.
Option B: Surgery is indicated if the cause of dysphagia is found to be cancer or Zenker’s diverticulum. Achalasia unresponsive to conservative measures may eventually need surgical intervention as well.
Option D: Helicobacter pylori can cause peptic ulcer and consequently strictures of the gastric outlet (more common) or inlet (less common). Stricture at the junction of the esophagus to the stomach may cause dysphagia, but difficulty in initiation of swallowing goes against this diagnosis.
Option E: Manometry can be used once barium meal study suggests a motility disorder such as achalasia.
A mother of two children aged 9 and 18 years is concerned about her children developing hemochromatosis, as his husband has recently been diagnoses with the disease. Regarding screening for hemochromatosis, which one of the following is the next best in management?
A. Screen the 9-year-old boy.
B. Screen the18-year-old boy.
C. Screen both children.
D. Screen the mother.
E. No screening is needed as the disease becomes clinically evident between 30-60 years.
D. Screen the mother.
The gene involved in hereditary hemochromatosis (HH) is called the HFE gene. Mutations in the HFE gene can lead to impaired regulation of iron storage and clinical manifestations of hemochromatosis. There are two types of mutation in HFE gene: C282Y and H63D.
Terminology
- Those with only one copy of the mutated HFE (either C282Y or H63D) gene are called heterozygote.
- Those with two copies of C282Y mutation are called ‘homozygote’. Since those with two copies of H63D mutation never develop clinical hemochromatosis, homozygote, refers to a person with both copies of HFE with C282Y mutation only.
- Those with one copy of HFE gene with C282Y mutation and the other with H63D mutation are called ‘compound heterozygote’. These individuals are often asymptomatic and if symptoms are present they are mild.
About 90% of people of Northern European ancestry with symptoms of HH have the C282Y mutation in both copies of their HFE gene (homozygote). Two percent are compound heterozygote (see above).
Since HH follows an autosomal recessive pattern of inheritance, there is often no family history, or affected family members may appear to be scattered in generations. If both parents heterozygous (carriers for a mutation in the HFE gene), there is a 25% chance for their children to be affected and genetically predisposed to HH.
Current guidelines advise that first-degree and second-degree relatives of individuals, who have HH or are homozygous for the C282Y gene mutation, are tested with iron studies and the HFE gene test. Based on this recommendation, both children should be considered for testing; however, in this case testing the mother would be more convenient and reasonable. If she does not have a faulty HFE gene, no further testing of the children would be indicated, because even if the father is homozygote, the children would only be carriers in the worst case scenario.
The photograph is one of a barium swallow series performed in a 78-year-old man. He has presented with symptoms of 12 months duration. Which one of the following could be the most likely presenting symptom?
A. Recurrent chest infection.
B. Progressive weight loss.
C. Retrosternal burning sensation.
D. Gurgling in the neck.
E. Food regurgitation.
E. Food regurgitation.
The photograph shows a pocket of contrast material at the root of the neck, as well as the contrast in the esophagus characteristic of retropharyngeal pouch (Zenker’s diverticulum). The condition is most commonly found in the elderly population.
Patients with Zenker’s diverticulum usually have dysphagia because the primary problem is an overactive upper esophageal sphincter, which fails to relax. Despite long-standing dysphagia, patients usually do not have significant weight loss.
When the pharyngeal pouch becomes large enough to retain contents such as mucus, pills, sputum and food, the patient may complain of pulmonary aspiration and recurrent chest infections, foul-smell breath, gurgling in the throat, appearance of a mass in the neck, or regurgitation of food into the mouth.
Of these symptoms, however, food regurgitation is the most distressing symptoms for which medical attention is usually sought.
Retrosternal burning sensation is a characteristic feature of gastro-esophageal reflux disease (GERD) and is not an associated symptom in Zenker’s diverticulum.
AMC Handbook of Multiple Choice Questions – pages 449-450
A 69-year-old woman is diagnosed with carcinoma of the cecum. Which one of the following is more likely to have been her initial presenting symptoms?
A. Right iliac fossa (RIF) mass.
B. Altered bowel habit.
C. Weakness and fatigue.
D. Melena.
E. Bright rectal bleeding.
C. Weakness and fatigue.
Colorectal cancers present with different symptoms depending on their location within the colon:
- Right-sided tumors typically cause symptoms such as anemia and fatigue due to chronic blood loss. This is because the proximal colon (right side) has a larger lumen and can accommodate liquid feces, leading to less noticeable changes in bowel habits.
- Left-sided tumors, including those in the rectum, commonly present with altered bowel habits (such as constipation or diarrhea) and rectal bleeding. This is because the left side of the colon has a narrower lumen, which can lead to obstruction or changes in stool consistency, causing symptoms like constipation or diarrhea.
Understanding these differences helps in recognizing and diagnosing colorectal cancers based on the presenting symptoms associated with the tumor’s location in the colon.
Colorectal cancers may present with a wide variety of symptoms. The presenting symptoms, to a great extent, depends on the location of the tumor.
A change in bowel habits is a less common presenting symptom for right-sided tumors because feces is liquid in the proximal colon and the lumen caliber is larger. Right-sided tumors present with anemia and fatigue due to chronic blood loss, while tumors of the left side are associated with altered bowel habits and rectal bleeding.
Option A: If a right-sided tumor is large enough, a right iliac fossa mass may be palpated. But a tumor that large has already caused significant symptoms for which the patient has already sought medical attention.
Option B: Altered bowel habit is more commonly seen in left-sided colon cancers, including rectal tumors.
Option D: Melena is associated with upper gastrointestinal (GI) bleeding with prolonged passage time of the blood through the gastrointestinal tract. Colorectal tumors are very unlikely to cause melena.
Option E: Rectal bleeding is often caused by a rectal cancer or more distal left-sided colon cancers.
Which one of the following is the most useful investigation for detection of gallstones and dilatation of the common bile duct?
A. HIDA scintigraphy.
B. Ultrasound.
C. Endoscopic retrograde cholangio pancreatography (ERCP).
D. X-ray.
E. Liver isotope scan.
B. Ultrasound.
Ultrasound is the most useful investigation for detection of gallstones and dilatation of common bile duct. It is also useful in detection of hepatic metastases and some liver diseases.
Option A: HIDA scan is used if sonographic studies are equivocal. On scan, the liver, CBD, and duodenum light up; the gallbladder will not if inflamed.
Option C: ERCP is used for visual detection and retrieval of stones in the CBD as well as other causes of obstructive jaundice.
Option D: Since only 10% of gallstones are radio-opaque, abdominal X-ray will be able to pick up gallstones in only up to 10% of cases; hence, not a preferred method.
Option E: Liver isotopic scan is useful for evaluation of hepatic cirrhosis.
A 48-year-old male presents to your practice for evaluation of his liver disease. He has the past medical history of chronic alcoholism, intravenous drug abuse and hepatic cirrhosis. Laboratory studies show deranged liver function tests. Which one of the following would be the best indicator of chronic liver disease?
A. Alkaline phosphatase.
B. Albumin.
C. Alanine amino transferase(ALT).
D. Aspartate amino transferase(AST).
E. Bilirubin.
B. Albumin.
Albumin is synthesized in the liver and has a half-life of around 20-22 days. Of the given options, albumin is the only indicator of chronic liver disease.
Option A: Alkaline phosphatase is NOT specific to the liver and can be elevated in the following conditions:
- Paget’s disease
- Fractures
- Cholestasis (bile duct obstruction, cirrhosis, etc)
- Malignant diseases with bony metastasis
Option C and D: Alanine aminotransferase is specific to the liver and is raised in metabolic syndrome, obesity, fatty liver and liver failure. ALT and AST are indicators of hepatocellular damage and can be elevated in both acute (e.g. viral hepatitis) and chronic liver disease. Interestingly, ALT and AST may be normal until the very last stages of chronic liver disease.
Option E: Bilirubin can be elevated in both acute and chronic liver diseases.
You are one of the senior residents in surgery. You are called to see Mr. Kingsley, a 67-year-old man, who has just been diagnosed with acute cholangitis. Which one of the following statements is not correct regarding the management of acute cholangitis?
A. Plan for immediate decompression if the patient does not respond to initial measures.
B. Plan for biliary decompression on semi-urgent basis (< 72 hours) if the patient is responding to initial resuscitation.
C. Plan for urgent decompression (within 24-48hrs) if the patient is older than 70 years.
D. The most appropriate method of biliary decompression is ERCP, sphincterectomy and stenting.
E. Initial aggressive resuscitation and antibiotics usually fail to get good response in majority of cases.
E. Initial aggressive resuscitation and antibiotics usually fail to get good response in majority of cases.
Acute ascending cholangitis is initially managed with aggressive fluid resuscitation and intravenous antibiotics followed by biliary decompression.
Since the infectious organisms responsible for acute ascending cholangitis are enteric gram negative bacteria, the selected antibiotic of choice should provide appropriate coverage against these germs.
All patients with ascending cholangitis require biliary drainage. In about 85-90% of patients, there is respond to medical therapy. In this group decompression may be performed semi-electively during the same admission (and ideally within 72 hours); however for the following patients urgent decompression may be considered:
- Patients older than 70 years
- Patients with diabetes
- Patients with other comorbid conditions
Approximately 10% to 15% of patients (not the majority) fail to respond within 12 to 24 hours or deteriorate after initial medical therapy and need urgent biliary decompression. Delay to do so increases the chance of an adverse outcomes.
A 42-year-old man is admitted with a hematemesis. On examination he has a blood pressure of 95/60 mmHg and a pulse rate of 104 bpm. He has yellowish sclerae, scratch marks and bruises on his arms. Several spider nevi are noted on his chest and abdomen. His abdomen is distended and soft with evidence of shifting dullness. Dilated veins are easily visible in the subcutaneous tissue and the liver is palpable two finger-breadths below the costal margin. Which one of the following could be the most likely underlying cause of his problem?
A. Alcoholic liver disease.
B. Biliary cirrhosis.
C. Schistosomiasis.
D. Budd-Chiari syndrome.
E. Hepatitis C cirrhosis.
A. Alcoholic liver disease.
Upper gastrointestinal bleeding may have different causes, but I this man having the clinical features of liver disease and cirrhosis, bleeding from esophageal varices would be the most likely cause of bleeding. Cirrhosis results in portal vein hypertension.
When the portal vein is obstructed, shunting between the portal vein and systemic veins start to develop. This shunts lead to dilated veins around the umbilicus (caput medusa), in the lower gastrointestinal tract (hemorrhoids), or in the upper gastrointestinal tract (esophageal varices).
In Australia alcoholic liver disease is the most common cause of chronic liver disease and cirrhosis, followed by hepatitis C infection.
Biliary cirrhosis, schistosomiasis, and Budd-Chiari syndrome can all potentially lead to liver cirrhosis, but are much less common than alcoholic liver disease.
A 55-year-old man presents with a 6-month history of increasing dysphagia for solid foods. He has a previous history of gastroesophageal reflux for many years. He has managed his reflux with antacids but since the dysphagia started, his reflux has not been so troublesome. Which one of the following is the most likely diagnosis?
A. Esophageal cancer.
B. Achalasia.
C. Gatro-esophageal junction stricture.
D. Para-esophageal hernia.
E. Ulcerative esophagitis.
C. Gatro-esophageal junction stricture.
Dysphagia to solid food is more likely to be caused by mechanical obstruction due to strictures, tumors, rings or webs.
With the history of protracted gastro-esophageal reflux disease (GERD), the most likely cause of this presentation would be stricture. Inflammation and scarring of the esophagus result in stenosis of the esophagus most often at the site of junction to the stomach. Resolution of GERD symptoms supports the diagnosis: when the stricture develops, the amount of acid reflux is decreased.
Option A: Esophageal cancer is another important differential diagnosis that has to be excluded, but not the most likely diagnosis given the absence of other symptoms such as significant weight loss, anemia, etc.
Option B: Achalasia causes dysphagia to both solids and liquids. Dysphagia to solids, but not to liquids is against out achalasia as a probability diagnosis.
Option D: Para-esophageal hernias may present with GERD and GERD, over time, may result in this clinical picture. The stricture, however, is caused by the GERD, not directly by the hernia.
Option E: Ulcerative esophagitis is very rarely complicated by strictures, especially in patients with HIV; hence, an unlikely diagnosis in this HIV negative patient.
Esophageal Stricture
A 66-year-old man presents with intermittent right upper quadrant pain. An ultrasound, performed for revealing the cause gallstones, reveals a lesion in the liver. Triple phase CT scan is performed for more evaluation showing a 35 mm subcortical lesion with early prominent dense enhancement, which spreads through the lesion in the late portal venous phase. Which one of the following conditions would fit this description best?
A. Isolated metastatic lesion.
B. Hemangioma.
C. Hepatocellular carcinoma.
D. Hepatic cyst.
E. Hydatid cyst.
B. Hemangioma.
The patient has the provisional diagnosis of the biliary colic, most likely due to biliary stones. The ultrasound scan – as the best initial diagnostic tool –has been used to confirm the diagnosis. Furthermore, any associated inflammation would be evaluated. Inflammation of the gallbladder manifests as the thickening of the wall of the gallbladder and the presence of pericholecystic fluid. Stones might be seen in the common bile duct as well; however, the sensitivity of ultrasound for detection of ductal stones is low (30%-50%).
As a routine procedure, when scanning for biliary problems, the sonographer will scan the liver as well. In this case the sonographer has encountered an incidental finding, irrelevant to the presenting symptoms, for which a triphasic CT scan of the liver has been performed.
Early prominent dense enhancement of the lesion during the arterial phase is characteristic of liver hemangioma (the most common benign liver tumor). Hemangiomas are seen in approximately 20% of the general population. They may be solitary or multiple. The lesions typically show intense enhancement during the arterial phase of triphasic CT scan and retain a blush of contrast during the portal venous phase.
Option A and C: Most malignant liver tumours (primary or metastatic) are hypovascular and will not have the early enhancement during the arterial phase; rather, they become more pronounced during the portal venous phase.
Option D: Cystic lesions in the liver may be simple, multiple (polycystic liver disease), neoplastic or infective (hydatid cysts). Simple cysts are extremely common and usually asymptomatic. On imaging, these cysts have a low-density homogenous appearance. With polycystic disease, the number and size of the cysts often lead to symptoms.
Option E: Hydatid cysts have a characteristic septate appearance and heterogenous appearance if they contain daughter cysts. Liver abscesses are usually symptomatic and more likely to have a heterogenous appearance.
You are about to perform a femoral venepuncture and you should take precaution not to damage the adjacent structures. Which one of the following is the order of structures in the groin under the inguinal ligament from medial to lateral?
A. Lacunar ligament, femoral artery, femoral vein, femoral nerve.
B. Lacunar ligament, femoral vein, femoral nerve, femoral artery.
C. Lacunar ligament, femoral vein, femoral artery, femoral nerve.
D. Femoral vein, femoral artery, femoral nerve, lacunar ligament.
E. Femoral vein. Femoral artery, lacunar ligament, femoral nerve.
C. Lacunar ligament, femoral vein, femoral artery, femoral nerve.
Femoral triangle consists of three borders:
- Upper border: inguinal ligament
- Medial border: lateral border of adductor longus
- Lateral border: medial border of sartorius
The contents of the femoral triangle from medial to lateral are:
1. Lacunar ligament and deep femoral lymph nodes
2. Femoral vein
3. Femoral artery
4. Femoral nerve
Remember LEVAN: Lymph node chain / lacunar ligament, Empty space, Vein, Artery and Nerve for the order of contents from medial to lateral.
The lacunar ligament is a ligament in the inguinal region that connects the inguinal ligament to the pectineal ligament near the point where they both insert on the pubic tubercle. This ligament comprises the medial border of the femoral canal.
Which one of the following is the most common early complication of hemorrhagic pancreatitis?
A. Pseudocyst.
B. Infection.
C. Obstructive jaundice.
D. Pancreatic fistula.
E. Renal failure.
E. Renal failure.
Acute pancreatitis can be classified into acute interstitial (most common) and acute hemorrhagic (least common). In the first type, the gland architecture is preserved but is edematous. Inflammatory cells and interstitial edema are prominent within the parenchyma. In hemorrhagic type, there is marked necrosis, hemorrhage, and fat necrosis. There is marked pancreatic necrosis along with vascular inflammation and thrombosis.
Hemorrhagic pancreatitis can rapidly result in severe hemorrhage, hypovolemia, shock and acute renal failure. Other options are also potential complications of acute pancreatitis but often do not occur as early as acute renal failure.
A 57-year-old man presents to your practice complaining of abdominal discomfort and pain for the past 6 months. The pain is predominantly felt in the epigastric area. He does not smoke but admits to chronic alcohol use. On examination, no abdominal tenderness is elicited. The remainder of the exam is inconclusive. An abdominal ultrasound scan is arranged that reveals the presence of a 10 cm cystic lesion in the epigastric area. Which one the following is the most appropriate management option?
A. Endoscopic gastrostomy.
B. Laparotomy.
C. Percutaneous drainage.
D. Drainage through ERCP.
E. Conservative management and re-evaluation in 6 months.
D. Drainage through ERCP.
A pancreatic pseudocyst is a collection of pancreatic juice encased by reactive granulation tissue (not epithelial tissue) in or around the pancreas. Pseudocysts can be single or multiple, small or large, and can be located either within or outside the pancreas. Most pseudocysts communicate with the pancreatic ductal system and contain high concentrations of digestive enzymes such as amylase and lipase.
The walls of pseudocysts are formed by adjacent structures such as the stomach, transverse mesocolon, gastrocolic omentum, and pancreas. The lining of pancreatic pseudocysts consists of fibrous and granulation tissue; the lack of an epithelial lining distinguishes pseudocysts from true pancreatic cysts.
The mechanism by which a pseudocyst is formed is necrosis and liquefaction of the pancreatic necrosis of pancreatic or peripancreatic tissue.
Pseudocyst can be seen in the following situations:
- After an episode of acute pancreatitis (in 10% of patients) - necrosis of peripancreatic tissue progresses to liquefaction and pseudocyst formation. Alternatively, a pseudocyst may result from parenchymal necrosis leading to the complete ductal disruption, and gross leakage of pancreatic juice.
- In patients with chronic pancreatitis – pseudocysts may develop after acute attacks of pancreatitis or after the pancreatic duct is obstructed. The latter causes increased intraductal pressure and leakage of pancreatic juice.
- After blunt or penetrating abdominal trauma (including iatrogenic injuries) - The injury can directly disrupt the duct and causes leakage.
Clinical manifestations and complications of pancreatic pseudocysts include:
- Expansion of the pseudocyst can produce abdominal pain, duodenal or biliary obstruction, vascular occlusion, or fistula formation into adjacent viscera, the pleural space, or pericardium.
- Spontaneous infection.
- Digestion of an adjacent vessel can result in a pseudoaneurysm, which can produce a sudden, painful expansion of the cyst or gastrointestinal bleeding due to bleeding into the pancreatic duct.
- Pancreatic pleural effusion -can result from disruption of the pancreatic duct with fistulization into the chest.
- Pancreatic peritonitis - can be caused by disruption of the pancreatic duct with fistulisation to the abdomen.
NOTE - up to 40% of pseudocysts resolve without intervention; however, they can produce a wide range of clinical problems depending on the location and extent of the fluid collection and the presence of infection.
Pancreatitis pseudocysts are diagnoses with CT or ultrasound scan. Where the diagnosis is in doubt, the content can be aspirated (under endoscopic ultrasonography or CT scan) and examined.
The old rule mentioning that intervention is needed if the cysts are larger than 6 cm or persist beyond 6 weeks is no longer in use, and surgical intervention should be considered if any of the following is present:
- Compression of large vessels (clinical symptoms or seen on CT scan)
- Gastric or duodenal outlet obstruction
- Stenosis of the common bile duct due to compression
- Infected pancreatic pseudocysts
- Hemorrhage into pancreatic pseudocyst
- Pancreatico-pleural fistula
- Pancreatic pseudocysts and symptoms:
— Early satiety
— Nausea and vomiting
— Pain
— Upper gastrointestinal bleeding - Asymptomatic pancreatic pseudocyst AND either of the following:
— Pseudocysts > 5cm, unchanged in size and morphology for more than 6 weeks
— Diameter > 4cm and extrapancreatic complications in patients with chronic alcoholic pancreatitis
— Suspected malignancy
Surgical drainage is the criterion standard against which all other interventions are measured in terms of success rate, mortality and recurrence rate. In recent years, however, endoscopic drainage has been introduced and can be applied provided that the cyst is near the stomach or duodenal wall:
There are two main types of endoscopic drainage:
- Transmural drainage: in this method, using endoscopy, a small incision is made in the stomach (endoscopic cystgastrostomy [ECG]) or in duodenum (endoscopic cystduodenostomy [ECD]) to let the pseudocyst drain into the stomach or duodenum. ECD is preferred over ECG.
NOTE - pseudocysts should have a mature capsule (wall thickness>3mm and < 1cm), bulge the lumen and have minimum size of 5-6 cm to become eligible for endoscopic drainage.
- Transpapillary drainage: this method is safer and more effective than transmural drainage, but requires that the cyst communicates with the pancreatic duct because this method includes entering the pancreatic duct by ERCP, and from there, into the pseudocyst. Stents may be left in place to facilitate drainage.
Generally, endoscopic drainage methods are preferred over open surgical treatment if eligibility is met and there is no contraindication because these methods are less invasive and associated with fewer complications.
Laparotomy with cyst excision and internal and external drainage is still the gold standard management option; however it is ONLY considered first-line therapy for surgical intervention in the following conditions:
- Complicated pseudocysts i.e. infected or necrotic
- Pseudocysts associated with pancreatic duct stricture and a dilated pancreatic duct
- Suspected cystic neoplasia
- Presence of pseudoaneurysm, unless it has been embolised before the procedure
- Coexistence pseudocysts and bile duct stenosis
- Complications such as compression of the stomach or the duodenum, perforation or pseudoaneursyms
NOTE - pseudoaneurysm is an absolute contraindication to endoscopic drainage unless it is embolized prior to the procedure.
Generally, patients with symptomatic pseudocysts should undergo interventional measures for pseudocyst drainage. The procedure of choice is endoscopic drainage. This cyst is 10 cm in size (>5cm) and amenable to endoscopic drainage either by endoscopic transmural or transpapillary drainage. Transpapillary drainage has the lowest complication rate of all the mentioned procedures and is the method of choice if the pseudocyst communicates with the pancreatic duct. Fortunately, 80% of pseudocysts communicate with the pancreatic duct.
Option A: Endoscopic cystgastrostomy (ECG) or duodenostomy (ECD) are methods of choice if the pseudocyst is not communicating with the pancreatic duct.
Option B: Laparotomy and surgical removal of the cyst is considered if endoscopic methods fail or there is a contraindication.
Option C: Percutaneous catheter drainage has low success rate and high recurrent rates. It is never considered for treatment of a pancreatic pseudocyst. However, in infected pseudocysts it is the procedure of choice for sampling and examining the material as the most appropriate initial step.
Option E: Conservative management is not an appropriate option for symptomatic pseudocysts.
Pancreatic Pseudocysts
A 34-year-old alcoholic man, who survived an episode of acute pancreatitis 4 weeks ago, has presented with mild discomfort in the epigastrium for the past few days. He is concerned that the disease might have recurred. He denies nausea and vomiting and describes the pain as constant and nagging. Physical examination is unremarkable. A contrast abdominal CT scan is performed showing a 4-cm pancreatic pseudocyst. Which one of the following is the next best step in management?
A. Endoscopic decompression.
B. Open surface decompression.
C. Observation.
D. Percutaneous catheter drainage.
E. Urgently take him to the operating room.
C. Observation.
A pancreatic pseudocyst is a collection of pancreatic juice encased by reactive granulation tissue (and not epithelial tissue) in or around the pancreas. Pseudocysts can be single or multiple, small or large, and can be located either within or outside of the pancreas. Most pseudocysts communicate with the pancreatic ductal system and contain high concentrations of digestive enzymes such as amylase and lipase.
The walls of pseudocysts are formed by adjacent structures such as the stomach, transverse mesocolon, gastrocolic omentum, and pancreas. The lining of a pancreatic pseudocysts consists of fibrous and granulation tissue. Lack of an epithelial lining distinguishes pseudocysts from true pancreatic cysts.
The mechanism by which a pseudocyst is formed is necrosis and liquefaction of the pancreatic necrosis of pancreatic or peripancreatic tissue.
Pseudocyst can be seen in the following situations:
- After an episode of acute pancreatitis (in 10% of patients) - necrosis of peripancreatic tissue progresses to liquefaction and pseudocyst formation. Alternatively, a pseudocyst may result from parenchymal necrosis leading to the complete ductal disruption, and gross leakage of pancreatic juice.
- In patients with chronic pancreatitis - pseudocysts may develop after acute attacks of pancreatitis or after the pancreatic duct is obstructed. The latter causes increased Intraductal pressure and leakage of pancreatic juice.
- After abdominal blunt or penetrating trauma (including iatrogenic injuries such as pancreatic surgery - injury can directly disrupt the duct and causes leakage.
Clinical manifestations and complications of pancreatic pseudocysts include:
- Expansion of the pseudocyst can produce abdominal pain, duodenal or biliary obstruction, vascular occlusion, or fistula formation into adjacent viscera, the pleural space, or pericardium.
- Spontaneous infection.
- Digestion of an adjacent vessel can result in a pseudoaneurysm, which can produce a sudden, painful expansion of the cyst or gastrointestinal bleeding due to bleeding into the pancreatic duct.
- Pancreatic pleural effusion -can result from disruption of the pancreatic duct with fistulization to the chest.
- Pancreatic pleural effusion: can be caused by disruption of the pancreatic duct with fistulization to the abdomen.
NOTE - up to 40% of pseudocysts resolve without intervention; however, they can produce a wide range of clinical problems depending on the location and extent of the fluid collection and the presence of infection.
Pancreatitis pseudocysts are diagnoses with CT or ultrasound scan. If the diagnosis is in doubt, contents can be aspirated and examined.
The dictum mentioning that intervention is needed if the cysts are larger than 6 cm or persist beyond 6 weeks is no longer in use, and surgical intervention should be considered in (even one criterion is sufficient):
- Compression of large vessels (clinical symptoms or seen on CT scan)
- Gastric or duodenal outlet obstruction
- Stenosis of the common bile duct due to compression
- Infected pancreatic pseudocysts
- Hemorrhage into pancreatic pseudocyst
- Pancreatico-pleural fistula
- Pancreatic pseudocysts and symptoms:
— Early satiety
— Nausea and vomiting
— Pain
— Upper gastrointestinal bleeding - Asymptomatic pancreatic pseudocyst AND either of the following:
— Pseudocysts > 5cm, unchanged in size and morphology for more than 6 weeks
— Diameter > 4cm and extrapancreatic complications in patients with chronic alcoholic pancreatitis
— Suspected malignancy
Surgical drainage (laparotomy and internal and external drainage) is the criterion standard against which all other interventions are measured in terms of success rate, mortality and recurrence rate. In recent years however endoscopic drainage has been introduced and can be applied provided that the cyst is near the stomach or duodenal wall:
There are two main types of endoscopic drainage:
- Transmural drainage: in this method, using endoscopy, a small incision is made in the stomach (endoscopic cystgastrostomy [ECG]) or in duodenum (endoscopic cystduodenostomy [ECD]) to let the pseudocyst drain into the stomach or duodenum. ECD is preferred over ECG.
- Transpapillary drainage: this method is safer and more effective than transmural drainage, but requires that the cyst communicates with pancreatic duct. Using ERCP and through the pancreatic duct, the cyst is reached and poked so that its content can drain into the pancreatic duct. Stents may be used to facilitate drainage.
NOTE - pseudocysts should have a mature capsule (wall thickness>3mm and <1cm), bulge the lumen and have minimum size of 5-6 cm to become eligible for endoscopic drainage.
Generally, endoscopic drainage methods are preferred over open surgical treatment if eligibility is met and there is no contraindication, because these methods are less invasive and associated with fewer complications.
Laparotomy with cyst excision and internal and external drainage is still the gold standard management option; however it is considered first-line therapy for surgical intervention if:
- Complicated pseudocysts i.e. infected and necrotic pseudocysts
- Pseudocysts associated with pancreatic duct stricture and a dilated pancreatic duct
- Suspected cystic neoplasia
- Presence of pseudoaneurysm unless it has been embolized before the procedure
- Coexistence pseudocysts and bile duct stenosis
- Complications such as compression of the stomach or the duodenum, perforation or pseudoaneursyms
NOTE - pseudoaneurysm is an absolute contraindication to endoscopic drainage unless it is embolized prior to the procedure.
Generally, patients with symptomatic pseudocysts should undergo interventional measures for pseudocyst drainage. The procedure of choice is endoscopic drainage, but as mentioned before, for a pseudocyst to be amenable to this procedure, it has to be at least 5-6 in size as well as bulging into the lumen
. This cyst with 4 cm in size is unlikely to be drained by endoscopic measures; therefore, open decompression (surgical drainage) should be considered; however, since the pain is mild, a watchful observation would be best management here in an attempt to avoid the high rates complications associated with surgical drainage. If the symptoms were more pronounce, intolerable or indicative of more serious complications, surgical drainage (open surface drainage) would have been the option of choice.
Percutaneous catheter drainage has low success rate and high recurrent rates. It is never considered for treatment of a pancreatic pseudocyst. However, in infected pseudocysts it is the procedure of choice for sampling and examining the material as the most appropriate initial step.
A 51-year-old Aboriginal male with history of acute pancreatitis presents with persistent abdominal pain and loss of appetite. On examination, an abdominal mass is found. A contrast abdominal CT scan is performed one cut of which shown in the following photograph. Which one of the following is the most likely diagnosis?
A. Chronic pancreatitis.
B. Acute pancreatitis.
C. Pancreatic pseudocyst.
D. Gastric adenocarcinoma.
E. Gastric lymphoma.
C. Pancreatic pseudocyst.
The round lesion with hypodense homogenous content and the thin smooth wall in the vicinity of and obliterating the pancreas is highly suggestive of a pancreatic cyst or pseudocyst. With the history of previous acute pancreatitis, a pseudocyst would be the most likely diagnosis.
A pancreatic pseudocyst is a collection of pancreatic juice encased by reactive granulation tissue (and not epithelial tissue) in or around the pancreas. Pseudocysts can be single or multiple, small or large, and can be located either within or outside of the pancreas. Most pseudocysts communicate with the pancreatic ductal system and contain high concentrations of digestive enzymes such as amylase and lipase.
The walls of pseudocysts are formed by adjacent structures such as the stomach, transverse mesocolon, gastrocolic omentum, and the pancreas. The lining of pancreatic pseudocysts consists of fibrous and granulation tissue. Lack of an epithelial lining distinguishes pseudocysts from true pancreatic cysts.
The mechanism by which a pseudocyst is formed is necrosis and liquefaction of the pancreatic necrosis of pancreatic or peripancreatic tissue.
Pseudocyst can be seen in the following situations:
- After an episode of acute pancreatitis (in 10% of patients) - necrosis of peripancreatic tissue progresses to liquefaction and pseudocyst formation. Alternatively, a pseudocyst may result from parenchymal necrosis leading to the complete ductal disruption, and gross leakage of pancreatic juice.
- In patients with chronic pancreatitis - pseudocysts may develop after acute attacks of pancreatitis or after the pancreatic duct is obstructed. The latter causes increased Intraductal pressure and leakage of pancreatic juice.
- After abdominal blunt or penetrating trauma (including iatrogenic injuries such as pancreatic surgery - injury can directly disrupt the duct and causes leakage.
Clinical manifestations and complications of pancreatic pseudocysts include:
- Expansion of the pseudocyst can produce abdominal pain, duodenal or biliary obstruction, vascular occlusion, or fistula formation into adjacent viscera, the pleural space, or pericardium
- Spontaneous infection
- Digestion of an adjacent vessel can result in a pseudoaneurysm, which can produce a sudden, painful expansion of the cyst or gastrointestinal bleeding due to bleeding into the pancreatic duct
- Pancreatic pleural effusion -can result from disruption of the pancreatic duct with fistulisation to the chest
- Pancreatic peritonitis - can be caused by disruption of the pancreatic duct with fistulisation to the abdomen
A 45-year-old man presents with signs and symptoms of gastro-esophageal reflux disease (GERD) and is started on a course of omeprazole 20 mg BID for 6 weeks with complete resolution of the symptoms. Because of long-standing symptoms, an upper endoscopy is performed. The endoscopic view of the distal esophagus is shown in the following photograph. Histopathology shows intestinal metaplasia, but no dysplasia. Which one of the following is most appropriate next step in management?
A. Pneumatic dilation.
B. Continue PPIs with doubled dose.
C. Endoscopy after 24 months.
D. Endoscopy after 6 months.
E. No intervention is needed.
C. Endoscopy after 24 months.
The photograph shows Barrett’s esophagus. Barrett’s esophagus refers to metaplasia of the squamous epithelium of the distal esophagus to columnar epithelium of the intestinal type. The metaplasia is believed to be due to GERD in a susceptible individual.
Barrett’s esophagus is often associate with severe reflux disease, esophagitis, or strictures, but is also seen in asymptomatic or minimally symptomatic patients.
Barrett’s esophagus is a premalignant condition with a 40- to 125-fold increase in the incidence of adenocarcinoma of the lower esophagus; however, the absolute risk is relatively small and approximately 0.5% per year.
Every patient with the alarming symptoms of weight loss, anemia, heme-positive stool, and dysphagia, or GERD for more than 5 years should undergo endoscopy and biopsy. Further management depends on the histological findings and is summarized in the following:ENDOSCOPIC FINDINGS - MANAGEMENT
- Barrett’s esophagus (metaplasia) - Repeat endoscopy every 2-3 years for lesions ≥ 3cm and every 3-5 years for lesions < 3 cm.
- Low-grade dysplasia - Repeat endoscopy in 6 months.
- High grade dysplasia - Distal esophagetomy.
This patient has metaplastic changes involving an area of more than 3cm; therefore, the next step in management would be repeating the endoscopy in 2-3 years.
Option A: Pneumatic dilation is not a management option for GERD or Barrett’s esophagus.
Option B: Although proton pump inhibitors (PPIs) may be associated with the appearance of squamous islands and possibly some degree of regression, there is no evidence that PPIs reduce the incidence of esophageal cancer in patient’s with Barrett’s esophagus.
Option D: Endoscopy after 3-6 months (current recommendations: every 6 months) was the choice of management if low-grade dysplasia (not metaplasia) would be found on histological studies.
Option E: This patient needs close surveillance and follow-up and action should be taken for early detection of esophageal malignancies.
A 39-year-old Aboriginal man presents to your practice with a 6-week history of abdominal pain, nausea, vomiting and bowel motions that are difficult to flush down the toilet. The abdominal pain is described as a constant and disabling radiating to back. The pain is neither brought up, nor alleviated by eating. He is a chronic alcoholic and continues to drink alcohol in large amounts. Which one of the following investigations is most likely to reveal the underlying cause of his symptoms?
A. Investigations for exocrine function of pancreas.
B. Stools exam.
C. Transabdominal ultrasound.
D. ERCP.
E. Plain abdominal X-ray.
C. Transabdominal ultrasound.
The clinical picture and history of excessive alcohol use makes chronic pancreatitis the most likely diagnosis. Chronic pancreatitis presents with epigastric pain as the most dominant feature. The natural history of pain in chronic pancreatitis is highly variable. Most patients experience intermittent attacks of pain at unpredictable intervals, while a minority of patients have chronic pain. In most patients, pain severity either decreases or resolves over 5-25 years. In alcohol-induced disease, alcohol cessation may reduce the severity of pain.
The pain is constant, often radiates to back and may be associated with nauseas and vomiting. It is often felt in the epigastric area but may be felt on the left side or even right side. The pain may or may not be related to eating. If related, there may be weight loss due to fear of eating. At times of pain patients bend forward to the so-called “pancreatic position” or lie in the knee-chest position on their right or left side to decrease the pain intensity.
Patients with severe pancreatic exocrine dysfunction cannot properly digest complex foods and absorb partially digested breakdown products. Nonetheless, clinical significant protein and fat deficiencies does not develop until more than 90% of the pancreatic function is lost. However, fear of eating due to pain brought up by eating, can result in early weight loss. With severe chronic pancreatitis, insulin deficiency and diabetes mellitus may also develop.
Abdominal CT scan, MRI, ultrasound, plain X-ray films and ERCP can be used for diagnosis, but CT scan is the best initial test used for imaging studies when chronic pancreatitis is suspected. CT scans are 75-90% sensitive and 85% specific. Ultrasound is the second-line initial imaging study with a sensitivity of 60-70% and specificity of 80-90% and the best among option in the absence of CT scan.
Calcifications within the pancreatic duct are present on plain films in approximately 30% of cases, making palne films a less desirable option.
The characteristic finding on imaging is pancreatic duct calcifications, ductal dilation, enlargement of the pancreas and fluid collection.
Calcium deposition is most commonly seen in alcoholic pancreatitis, but is also present hereditary and tropical forms of the disorder. It is rare in idiopathic pancreatitis.
Magnetic resonance cholangiopancreatography (MRCP) is becoming the diagnostic test of choice since it can show calcification, without any radiation risks, but is expensive and not readily available. For this reason, it is not an ideal option for initial assessment.
Endoscopic retrograde cholangiopancreatography (ERCP) is reserved for situations where non-invasive modalities are not available, are equivocal and for intervention.
A 45-year-old man presents to the emergency department with acute onset epigastric and right upper quadrant abdominal pain that he describes as constant and severe. He is nauseous and has vomited 3 times since the pain started. He admits to chronic heavy alcohol consumption. On examination, he has a blood pressure of 140/90mmHg, pulse rate of 110bpm and temperature of 38.4°C. The epigastric area is mildly tender to palpation. Which one of the following is the most likely diagnosis?
A. Acute cholecystitis.
B. Acute pancreatitis.
C. Acute gastritis.
D. Acute gastro-esophageal reflux disease.
E. Alcohol intoxication.
B. Acute pancreatitis.
The clinical picture and the history of heavy alcohol use suggest acute pancreatitis as the most likely diagnosis.
The cardinal symptom of acute pancreatitis is abdominal pain. The pain is characteristically described as dull, boring and constant. The pain is often sudden-onset and gradually becomes worse. Most often, the pain is felt in the upper abdomen usually in the epigastric region, but sometimes is perceived more on the right or left side, depending on which part of the pancreas is involved. In about 50% of the patients, the pain directly radiates through the abdomen to the back. The duration of pain is variable but typically lasts more than 1 day.
Restless and agitation may be noted. At times of pain patients bend forward to the so-called “pancreatic position” or lie in the knee-chest position. Anorexia, nausea and vomiting, and diarrhea are other likely symptoms. Acute pancreatitis secondary to alcohol, frequently occurs 1 to 3 days after a binge drinking.It can also occur after cessation of drinking.
The patient should be asked about recent operative or other invasive procedures such as ERCP, family history of hypertriglyceridemia, previous biliary colic and binge alcohol drinking as the major causes of acute pancreatitis.
Option A: Acute cholecystitis causes pain in right upper quadrant, fever and leukocytosis. Murphy’s sign is usually positive: the patient is asked to inspire deeply while the right subcostal area is palpated. The Murphy sign is cessation of inspiration due to pain. Patients with acute cholecystitis may experience increased discomfort and hold in mid-inspiration. Abdominal examination may show voluntary and involuntary guarding.
Option B: Acute gastritis usually can present with pain in epigastrium associated with nauseas and vomiting and epigastric tenderness but fever would not be a feature.
Option D: Gastroesophageal reflux disease (GERD) presents with heartburn, chronic cough, a metalic taste in the mouth. The given clinical picture is completely different from that of GERD.
Option E: Patients with alcohol intoxication presents with slurred speech, nystagmus, disinhibited behavior, incoordination, unsteady gate, memory impairment, stupor, or coma depending on the severity of intoxication.
The following photograph is one cut from an abdominal CT scan in a 68-year-old man. Which one of the following is the most likely diagnosis?
A. Gastric outlet obstruction.
B. Cancer of the stomach.
C. Simple hepatic cyst.
D. Pancreatic pseudocyst.
E. Hepatic hemangioma.
A. Gastric outlet obstruction.
The picture shows a contrast CT scan of the abdomen. On the right side of the picture (left side of the patient) a distended contrast-filled stomach is seen. Duodenum is on the left side of the picture. It also contains contrast media.
The right kidney is another structure seen on the CT scan. In front of the vertebral body and slightly to the left, the aorta (containing contrast material) is noticed. Other structures seen on this cut of the CT scan are lower part of the right hepatic lobe, left hepatic lobe, spleen and parts of the pancreas.
The distended and fluid filled stomach is suggestive of gastric outlet obstruction (GOO). The distended duodenum indicates that the obstruction has occurred at the duodenum level.
Clinical entities that can result in GOO generally are categorized into two groups – benign and malignant. In the past, peptic ulcer disease (PUD) was the most common cause of GOO, but currently, 50-80% of GOOs are due to malignancies such as pylorus adenocarcinoma, lymphoma and gastrointestinal stromal tumour (GIST).
Benign causes of GOO include pancreatic pseudocysts, gastric varices, infections such as tuberculosis, and rarely gall stones.
At this level no pancreatic or hepatic lesion is seen. Cancer of the stomach would have given an obstruction in upper parts if it involves the pylorus. Besides, this might be the underlying cause of the CT scan findings not the interpretation.
A 35-year-old woman presents with a 4-month history of dysphagia to both solid and liquid foods. A barium study is done demonstrating dilated esophagus with tapering at the lower end. Based on the history and radiological findings, achalasia is the diagnosis. Which one of the following is the most appropriate management in this patient?
A. Pyloromyotomy.
B. Pneumatic dilation.
C. Botulinum toxin injection.
D. Calcium channel blockers.
E. Nitrates.
B. Pneumatic dilation.
Achalasia is a primary esophageal motility disorder characterized by the absence of esophageal peristalsis and impaired relaxation of the lower esophageal sphincter (LES) in response to swallowing. These abnormalities cause a functional obstruction
at the gastroesophageal junction.
The goal of treatment is to reduce the resistance of the LES and overcome the obstruction.
Currently, standard of care for achalasia is based on guidelines from the American College of Gastroenterology and is as follows:
- Initial therapy should be either graded pneumatic dilation (PD) or laparoscopic surgical myotomy in patients who are fit for such surgeries.
- Botulinum toxin injection into the LES is considered for patients who are not appropriate candidates for surgery.
- Pharmacologic therapy can be used for patients not undergoing PD or myotomy and who have failed botulinum toxin therapy (nitrates and calcium channel blockers most common)
For this patient, either pneumatic dilation or endoscopic myotomy (not an option) is the most appropriate initial management option.
Option A: Pyloromyotomy is the procedure used for treatment of hypertrophic pyloric stenosis or obstruction.
Option C: In patients with mild symptoms or in those who are not appropriate candidates for pneumatic balloon dilation or surgery, injection of botulinum toxin is considered the next most appropriate treatment. Botulinum toxin injection has been associated with symptoms improvement, but may need to be repeated at intervals of 3-12 months.
Option D and E: Pharmacological therapy can be used for those patients not undergoing pneumatic dilation or myotomy, and who have failed botulinum toxin therapy. These medications, however, may not be desirably effective. First-line medical treatment is with glyceryl trinitrate (oral or sublingual spray). If the episodes are associated with spasms or are frequent or disabling, a trial of diltiazem, nifedipine, or isosorbide dinitrate can be considered.
Achalasia
A 72-year-old woman presents to the emergency department with sudden-onset severe pain in the right lower quadrant and and back pain. She has the history of deep vein thrombosis (DVT) two months ago, for which she was started on warfarin. On examination, she has a blood pressure of 110/75 mmHg, pulse rate of 140 bpm and temperature of 37.2°C. There is a tender mass in the right iliac fossa (RIF). Plain X-ray of her abdomen is obtained and is shown in the following photograph. Which one of the following is the most likely diagnosis?
A. Abscess of the appendix.
B. Rectus sheath hematoma.
C. Cecal volvulus.
D. Leaking abdominal aortic aneurysm.
E. Fecal impaction.
C. Cecal volvulus.
The X-ray shows an extensively air-filled dilated proximal colon arising from the right side and extending to the left. With these X-ray findings, along with the history and physical findings, cecal volvulus is the most likely diagnosis.
Volvulus occurs when a segment of viscus twists around its axis and results result in obstruction as well as blood supply compromise.
The most common types of volvulus are sigmoid volvulus (more common) and cecal volvulus (less common). Cecal volvulus tends to occur in younger patients (30-60 years) compared to sigmoid volvulus that often is seen in elderly patients, who are bedridden or nursing home residents.
Option A: There are no suggestive clues to acute appendicitis as a precipitating event for an appendix abscess to form.
Option B: Although the clinical findings and history of warfarin use can make rectus sheath hematoma a likely diagnosis, the abdominal X-ray is consisetent with csecal volvulus.
Option D: With leaking abdominal aortic aneurysm, a tender pulsatile mass would be expected in the midline, not in the right iliac fossa.
Option E: With fecal impaction, fecal material was expected to be seen in colon (left side). Focal tenderness is not a dominant feature in fecal impaction.
A 65-year-old man is being evaluated for assessment of the source of gastrointestinal bleeding after he presented with complaint of melena at several occasions and was found to be anemic. Due to the very low level of hemoglobin and symptoms such as severe weakness and chest pain on exertion, he received several transfusions; however, he was not able to maintain a satisfactory hemoglobin level. Upper and lower endoscopy and small bowel barium studies were performed failing to find a source of bleeding. Which one of the following is the next best step in management?
A. Capsule endoscopy.
B. Interventional angiography.
C. CT angiography.
D. Repeat endoscopy.
E. Radionuclide imaging.
C. CT angiography.
The case scenario is an example of obscure gastrointestinal tract (GIT) bleeding. Obscure GIT bleeding is defined as bleeding from the GIT that persists or recurs without an obvious etiology found on upper endoscopy and colonoscopy and radiologic evaluation of the small bowel (such as by small bowel follow-through or enteroclysis).
Obscure GIT bleeding accounts for approximately 5% of patients with GIT bleeding. In 75% of these patients, the source of bleeding is in small bowel. The remainder of cases are due to missed lesions in either upper or lower GI tract.
Obscure bleeding is subdivided into overt or occult, depending upon the presence or absence of clinically-evident bleeding:
Obscure occult (inactive) bleeding – This is manifested as iron deficiency anemia or recurrent positive Fecal Occult Blood Test (FOBT) results.
Obscure overt (active) bleeding – This is manifested as recurrent episodes of clinically evident bleeding (e.g., melena or hematochezia), or persistent blood loss. Patients with ongoing blood loss so significant to necessitate transfusion are considered to have life-threatening active bleeding.
Further management of obscure GI bleeding depends on whether the bleeding is clinically active (overt) or inactive (occult):
Active obscure GI bleeding:
In patients presenting with active bleeding, radiological investigations are the mainstay of diagnostic approach, given their high sensitivity and non-invasiveness. CT Angiography (CTA) can detect bleeding rates of 0.5 ml/min and above. A negative CTA signifies that catheter angiography is not indicated since the latter is less sensitive than the CTA and has been shown to detect the source of bleeding only if the rate of blood loss is greater than 0.5 ml/min.
With a negative CTA and continuous bleeding, technetium 99m – labeled RBC nuclear scan is used as the second-line investigation at some institutions while others may use this as the first-line investigation. Among available modalities, RBC nuclear scan is most sensitive for active GI bleeding and can detect bleeding rates as low as 0.1 mL/min.
NOTE - Life-threatening hemorrhage should indicate catheter angiography as first-line management. A hemorrhage so significant is very likely to light up on catheter angiography. With catheter angiography, the site of bleeding can be found and controlled at the same time.
Inactive obscure GI bleeding:
Capsule endoscopy is the option of choice. However, a negative test demands CT angiography or other radiologic modalities (radionuclide scan, etc.) for further assessment.
—–
This patient has low hemoglobin despite several transfusions; however, he does not have urgent life-threatening hemodynamic instability such as low blood pressure, tachycardia, shortness of breath, chest pain, or hypoperfusion-related altered mental state. For him a CTA would be the most appropriate next step in management to find the source of bleeding.
Option A: Capsule endoscopy is the first option for patients with occult obscure GI bleeding i.e. iron deficiency anemia or recurrent positive Fecal occult blood test (FOBT) results.
Option B: Interventional angiography was the option of choice if the patient had immediate life-threatening hemodynamic instability at the moment. Although this patient has low hemoglobin, more transfusion can temporarily stabilize him to fulfill the criteria for CTA.
Option D: Repeating the endoscopic studies is unlikely to add any information regarding the site of bleeding, as this measure has already failed to do so.
Option E: Radionuclide imaging is used when there is obscured GI bleeding of unknown origin despite CT angiography or capsule endoscopy. The test takes a long time to perform. This patient with brisk and significant blood loss is not a good candidate for this test.
Which one of the following is the most common presenting symptom of a patient with duodenal stricture secondary to duodenal ulcer?
A. Vomiting 3-4 times a day.
B. Abdominal distension.
C. Right upper quadrant pain.
D. Vomiting after one hour of meals.
E. Vomiting early in the morning.
D. Vomiting after one hour of meals.
Gastric outlet obstruction (GOO), is the consequence of any disease producing a mechanical obstruction to gastric emptying. Clinical entities that can cause GOO are generally categorized into benign and malignant.
Peptic ulcer disease (PUD) is among the benign causes of GOO; however, the incidence of ulcer-induced GOO has dramatically declined owing to adequate and efficient treatment of PUD. The mechanism of obstruction by PUD can be either edema around the ulcer, or scar formation after the ulcer heals.
The most common symptoms of GOO, regardless of the underlying etiology, are bloating, anorexia, nausea and vomiting. Vomiting usually is described as nonbilious, and characteristically containing undigested food particles.
Patients with gastric outlet obstruction from a duodenal ulcer or incomplete obstruction typically present with symptoms of gastric retention, including early satiety, bloating or epigastric fullness, indigestion, anorexia, nausea, vomiting, epigastric pain, and weight loss. They are frequently malnourished and dehydrated and have a metabolic insufficiency. Weight loss is frequent when the condition is chronic and is most significant in patients with malignant disease.
Abdominal pain is not frequent and usually relates to the underlying cause, e.g., PUD, pancreatic cancer.
The time of vomiting can suggest the site of obstruction. In pyloric obstruction (more proximal) the time of vomitus is usually within the first hour of eating, whereas in pyloric stenosis, or duodenal stenosis or obstruction (more distal) the vomiting occurs after one hour because normally it takes 45 minutes to 1 hour for the food to reach the duodenum.
Option A: If the patient has 3 or 4 meals a day he may vomit 3 to 4 times within one hour after each meal, but vomiting 3 to 4 times a day by itself is not a common presentation of gastric outlet obstruction.
Option C: Right upper quadrant abdominal pain is seen in hepatobiliary diseases and is not a characteristic feature of gastric outlet obstruction.
Option B: Abdominal distension is a feature of bowel obstruction, particularly obstruction of the large bowel.
Option E: Early morning vomiting is associated with raised intra-cranial pressure and pregnancy, and is not a feature of gastric outlet obstruction.
A 56-year-old man presents to the emergency department with acute-onset severe epigastric pain, fever, nausea and vomiting. Past medical history includes type 2 diabetes mellitus, cardiomyopathy, and hypertension. Abdominal ultrasound shows a stone in the bile duct, multiple gallstones in the gallbladder and a dilated common bile duct of 6mm. Laboratory studies are significant for elevated conjugated bilirubin, elevated white cell count, and a normal hemoglobin level. Liver function tests show a 6-time rise in alanine aminotransferase and aspartate aminotransferase. Lipase and amylase are also elevated. Which one of the following options is the most appropriate next step in management?
A. Abdominal CT scan.
B. ERCP.
C. MRCP.
D. Upper endoscopy.
E. Monitor liver function tests.
B. ERCP.
The clinical and laboratory picture suggests acute pancreatitis secondary to gallstones and common bile duct obstruction that has led to LFT abnormalities as well.
Gallstone pancreatitis necessitates specific therapeutic consideration in addition to conventional treatment of acute pancreatitis.
Multiple studies suggest that early endoscopic retrograde cholangiopancreatography (ERCP), within 24-48 hours, with papillotomy, or surgical intervention to remove bile duct stones is beneficial in patients with acute biliary pancreatitis and the presence of the following:
- Concomitant acute cholangitis
- Persisted common bile duct obstruction evident by worsening signs or symptoms of obstruction (clinical deterioration), or deteriorating liver function tests
Endoscopic papillotomy is typically accompanied by placement of a plastic biliary stent to decrease the risk of post-ERCP pancreatitis.
With increased liver function tests (LFTs), especially the elevated conjugated bilirubin, this patient should be considered to have concomitant cholangitis until proven otherwise. Considering the likelihood of acute cholangitis caused by CBD obstruction, this patient should undergo early ERCP as the most appropriate next step in management, after rehydration, analgesia, and antibiotics. LFTs are used to monitor the response to treatment.
An ultrasound scan (not an option) is the very first imaging investigation for patients with signs and symptoms of pancreatitis or pancreatic cancer if manifestations of biliary obstruction are also present. Otherwise, Abdominal CT scan is used; However, in the absence of clinical features of biliary obstruction, CT scan is the preferred diagnostic modality.
CT scan is the most important imaging test for the diagnosis of acute pancreatitis, its severity and potential complication Patients with clinical and biochemical features of acute pancreatitis. CT scan is indicated if:
- there is no improvement with conservative therapy
- complications are suspected
- other diagnoses than pancreatitis is suspected
Acute Pancreatitis
A 70-year-old man presents to the emergency department with acute epigastric pain, nausea and vomiting. He has the history of gallstones. Laboratory tests show moderate elevation of lipase, amylase, alanine aminotransferase (ALT) and aspartate aminotransferase (AST). Both conjugated and unconjugated bilirubin are within the normal range. Inflammatory markers are not elevated. Abdominal ultrasound reveals a small stone in the common bile duct (CBD), but CBD is not dilated. Which one of the following is the most appropriate next step in management?
A. Abdominal CT scan.
B. MRCP.
C. ERCP.
D. Intravenous fluids, analgesics and antiemetics.
E. Monitoring the liver function tests.
D. Intravenous fluids, analgesics and antiemetics.
This patient has clinical diagnosis of acute pancreatitis, most likely secondary to gallstones; however, a non-dilated common bile duct (CBD) and normal bilirubin excludes CBD obstruction. Management of the acute pancreatitis should be started with intravenous fluids, analgesia and antiemetics. The patient should be nil by mouth (NPO) and NG tube inserted for GI decompression.
Further attention then should be paid to biliary stones as the underlying cause. Early ERCP (within 24-48 hours) is recommended for patients with acute biliary pancreatitis and either of the following:
* Concomitant acute cholangitis
* Persisted common bile duct obstruction evident by worsening sign or symptoms of obstruction, or deteriorating liver function tests
Elevated liver enzymes and/or CBD stone on ultrasound are pointers towards a biliary cause for acute pancreatitis; however, normal level of bilirubin and the non-dilated of CBD on ultrasound exclude cholangitis with high certainty. For this patient early ERCP is not indicated, but ideally the patient should undergo semi-elective cholecystectomy during this hospital stay.
CT scan is the most important imaging test for diagnosis of acute pancreatitis and its intraabdominal complications and also for assessment of severity. Patients with clinical and biochemical features if acute pancreatitis, who do not improve with initial conservative therapy or who are suspected of having complications or other diagnoses should undergo CT scan of the abdomen. This patient may need CT scan later (usually not earlier than 72 hours) in the course of the disease but not now as the most appropriate next step.
Monitoring LFTs are required for assessment of response to treatment, but not a priority in initial management.
A 49-year-old man with smoking history of 30 pack/year and longstanding gastro-esophageal reflux disease (GERD) presents with progressive difficulty in swallowing for the past 6 months. Initially, he had difficulty swallowing solid foods, but later he also developed dysphagia to liquids. He has malaise, fatigue, and has lost 15 kg in the past 3 months. On clinical examination, he looks pale and anemic. Which one of the following would be the next best step in management?
A. CT scan of the chest.
B. Barium Swallow.
C. Esophagoscopy.
D. Bronchoscopy.
E. Chest X-ray.
C. Esophagoscopy.
The clinical picture including the history of heavy smoking and GERD, as well as the characteristics of the dysphagia is suggestive of mechanical dysphagia most likely caused by esophageal cancer. Weight loss can be seen in any kind of dysphagia; however, a significant weight loss of 15 kg in only 3 months suggests a more malignant etiology. Anemia is another clue to malignancy.
When esophageal cancer is suspected, the best initial test is endoscopy (esophagoscopy) and biopsy for definite diagnosis.
Option B: Barium swallow is very sensitive for detection of strictures and intraluminal masses but does not allow staging and biopsy. It is now rarely used when cancer is suspected.
Option A, D and E: Bronchoscopy , chest X-ray , and CT scan of the chest are all used as tools for further evaluation and staging of the tumor, once esophagoscopy established a diagnosis of esophageal cancer.
Bronchoscopy may be used for malignancies of upper (and middle) esophagus because it is common for tumors of these parts to invade the tracheobronchial tree.
NOTE - The extent of tumor spread to the mediastinum and para-aortic lymph nodes should be assessed by CT scanning of the chest and abdomen and endoscopic ultrasound. Endoscopic ultrasonography is the most sensitive test for assessment of the tumor depth. Other studies to consider for staging are bone scan and liver function tests.
Which one of the following factors has the greatest impact on development of esophageal adenocarcinoma?
A. Achalasia.
B. Barrett’s esophagus.
C. Low-fiber diet.
D. Smoking.
E. Gastro-esophageal reflux disease.
B. Barrett’s esophagus.
While smoking and alcohol are the two most common risk factors for squamous cell carcinoma of the esophagus, Barrett’s esophagus is the most significant risk factor for development of esophageal adenocarcinoma.
Option A: Achalasia has been associated with esophageal cancer but is not as significant as other factors such as smoking and alcohol in squamous cell carcinoma and Barret’s esophagus in adenocarcinoma of the esophagus.
Option C: Although implicated in some studies, low-fiber diet is not as significant as other risk factors for esophageal cancer.
Option D: Smoking and chronic alcohol ingestion are the most common and important risk factors for development of esophageal squamous cell carcinoma.
Option E: Gastro-esophageal reflux disease is not a risk factor for esophageal adenocarcinoma unless it results in Barrett’s esophagus (indirect effect).
NOTE - Obesity is considered another important risk factor, mostly because it predisposes to reflux disease. However, the outcome of these conditions – GERD and the consequent Barrett’s esophagus, remains the main and most significant risk factor through producing metaplastic changes that can progress to dysplasia and eventually cancer.
A 42-year-old woman presents to the emergency department with right sided abdominal pain, mild jaundice and fever. She has a blood pressure of 130/85 mmHg, pulse rate of 90 bpm and temperature of 38.1°C. She is admitted to the hospital. After 3 hours, she develops rigors and chills. On re-exam, her blood pressure has fallen to 90/60 mmHg, her pulse has risen to 110 bpm and her temperature is 39.5°C. Which one of the following is the most likely diagnosis?
A. Septic shock.
B. Choledocholithiasis.
C. Acute pancreatitis.
D. Acute cholecystitis.
E. Acute cholangitis.
E. Acute cholangitis.
The clinical picture is highly suggestive of acute cholangitis as the most likely diagnosis. This patient has signs and symptoms consistent with diagnosis of acute cholangitis. Fever is present in approximately 90% of patients. Abdominal pain and jaundice occurs in 70% and 60% of patients, respectively. The patient can become quickly septic and hypotensive with altered level of consciousness.
Option A: Septic shock can result from acute cholangitis, and is a clinical syndrome not a diagnosis.
Option B: Choledocholithiasis is the presence of gallstones within the common bile duct. A patient with uncomplicated common bile duct stones typically develops abdominal pain with mild liver function abnormalities. Fever is not a usual fever in uncomplicated patients. The two major complications associated with choledocholithiasis are acute cholangitis and acute pancreatitis.
Option C: In acute pancreatitis, the abdominal pain is typically accompanied by nausea and vomiting, which may persist for many hours. The patient is often restless. Jaundice is not a feature unless there is concomitant common bile duct (CBD) obstruction and cholangitis.
Option D: Acute cholecystitis is inflammation of the gallbladder, characterized by right upper quadrant pain and tenderness, fever, and leukocytosis. Gallstones are the most common etiology. Jaundice is not a feature of acute cholecystitis.
A 56-year-old man presents with complaints of 10 kg weight loss in the past three months and jaundice. He denies any abdominal pain, but he describes symptoms of early satiety. Recently, he has noticed dark urine and pale stool. On examination, he has icteric sclera and a palpable right upper quadrant mass that moves with respiration. No abdominal tenderness is elicited. Which one of the following is the most appropriate initial investigation?
A. Abdominal CT scan.
B. Abdominal ultrasonography.
C. Chest X-ray.
D. ERCP.
E. MRCP.
B. Abdominal ultrasonography.
The clinical presentation is highly suggestive of pancreatic head carcinoma. This is evident by painless obstructive jaundice and a distended gallbladder (the right upper quadrant mass that moves with respiration).
Pancreatic cancer can present with the following (percentages are from one recent study of 186 patients with pancreatic cancer):
* Malaise and fatigue – 86%
* Weight loss – 85%
* Anorexia – 83%
* Abdominal pain – 79%
* Epigastric pain – 71%
* Dark urine – 59%
* Jaundice – 56%
* Nausea – 51%
* Back pain – 49%
* Diarrhea- 44%
* Vomiting – 33%
* Steatorrhoea – 25%
* Thrombophlebitis – 3%
The most frequent signs are:
* Jaundice – 55%
* Hepatomegaly- 39%
* Right upper quadrant mass – 15%
* Cachexia – 13%
* Courvoisier’s sign (nontender but palpable distended gallbladder at the right costal margin) – 13%
* Epigastric mass – 9%
* Ascites – 5%
Of all pancreatic malignant tumors, most are found in the pancreatic head as the most common site. Pancreatic body and tale are second and third most common sites, respectively.
Initial imaging study for assessment of suspected pancreatic cancer depends on the presenting symptoms. For patients with jaundice, the initial imaging study is typically a transabdominal ultrasound. It has high sensitivity for detecting biliary tract dilation and determining the level of the obstruction. It also has high sensitivity (>95%) for detection of a mass in the pancreas. The sensitivity is lower for tumours < 3 cm.
Option A: Abdominal CT , however, is the preferred initial imaging modality in patients with epigastric pain and weight loss, but without jaundice. A contrsast-enhanced CT scan of the abdomen is alwasy considered for such patients.
Option C: Chest X-ray is not indicated for diagnostic imaging of pancreatic cancer.
Option D and E: ERCP and MRCP may be indicated in further assessment, in cases with obstructive jaundice.
Cholestatic Jaundice
A 32-year-old woman presents to the Emergency Department with abdominal pain and fever since yesterday, which now has localized in the pelvis. On examination, he has a blood pressure of 110/85 mmHg, pulse rate of 98 bpm, respiratory rate of 22 breaths per minute, and fever of 38.2°C. Palpation of the abdomen elicits no tenderness, rebound, or guarding, but there is mild tenderness on rectal exam. Which one of the following would be the next best step in management of this patient?
A. Laparoscopic drainage and appendectomy.
B. Open drainage.
C. CT scan of the pelvis and the abdomen.
D. Ultrasonography of the pelvis and the abdomen.
E. Intravenous antibiotics.
D. Ultrasonography of the pelvis and the abdomen.
The clinical picture is suggestive of pelvic appendicitis. The most common symptom of appendicitis is abdominal pain. Typically, symptoms begin as periumbilical or epigastric pain migrating to right lower quadrant of the abdomen. This pain migration is the most characteristic feature in the patient’s history. However, in up to 30% of the time, the appendix may be hidden from the anterior peritoneum by being in pelvic, retroileal or retrocolic (retroperitoneal retrocecal) position. The hidden position of the appendix may significantly alter the clinical manifestation of the appendicitis.
A patient with a pelvic appendix may show no abdominal signs, but rectal examination may cause tenderness in cul-de-sac. Additionally, an obturator sign (pain on passive internal rotation of the flexed right thigh) may be present in a patient with pelvic appendix.
With typical presentation of appendicitis, imaging studies are not necessary and the patient will be taken to the operating room for appendicectomy, but when diagnosis of appendicitis is not certain (such as in this case) imaging studies are required for confirmation.
CT scan, specially the technique of appendiceal CT, is the most accurate imaging study (more accurate than ultrasonography). Appendiceal CT gives the highest diagnostic yield. If appendiceal CT is not available, standard abdominal/pelvic CT with contrast remains highly useful and may be more accurate than ultrasonography.
However, because of concerns about patient exposure to radiation during CT scans, ultrasonography has been suggested as a safer primary diagnostic modality for appendicitis, with CT scanning used secondarily when ultrasounds are negative or inconclusive. A healthy appendix usually cannot be viewed with ultrasonography. When appendicitis occurs, the ultrasonogram typically demonstrates a noncompressible tubular structure of 7-9 mm in diameter. CT scan is the next step if ultrasound is non-diagnostic.
NOTE - Pelvic appendicitis may present with:
- An absence of abdominal wall rigidity and tenderness
- Tenderness in the rectovesical pouch and Pouch of Douglas on rectal examination
- Spasm of psoas muscle on right-hand-side
- Diarrhea due to rectal irritation by the inflamed appendix
- Frequency of micturition due bladder irritation by the inflamed appendix
- Hypogastric pain on flexing and internally rotating the hip - due to contact of inflamed appendix with obturator internus muscle
A 57-year-old man presents to the emergency department with absolute constipation for two days and ongoing vomiting for the past eight hours. On examination, the abdomen is not distended or tender. The rest of the exam is unremarkable except signs of mild dehydration. Gastrograffin studies confirm the presence of small bowel obstruction. Which one of the following is the most common cause of small bowel obstruction?
A. Postoperative adhesions.
B. Bowel cancer.
C. Crohn’s disease.
D. Hernias.
E. Hypokalemia.
A. Postoperative adhesions.
Post-operative adhesions are the most common cause of small bowel obstruction (SBO). The risk of developing an obstruction after surgery from postoperative adhesions is estimated to be 9% within the first year after abdominal surgery, 19% by 4 years, and 35% by 10 years.
The incidence of adhesive SBO seems to be higher in open surgeries than in laparoscopic surgeries. The cause for such difference is not clear, but is likely to be related to the degree of manipulation and trauma to the intraabdominal tissues.
Malignant tumors are the second most common cause of SBO, accounting for about 20% of cases. SBO has been described in as many as 42% of women with ovarian carcinoma and 28% of patients with colorectal carcinomas.
Hernias are the third leading cause of SBO and account for approximately 10% of all cases. Ventral and inguinal hernias are most commonly associated with obstruction; however, internal hernias, femoral, obturator and parastomal hernias also can contribute to SBO.
Other less common causes of small bowel obstruction include:
- Strictures e.g. caused by Crohn’s disease
- Small bowel tumors
- Trauma
- Intussusception
- Bezoars
- Gallstone ileus (pseudo-obstruction)
- Superior mesenteric artery syndrome
- Hypokalemia (pseudo-obstruction)
Small-Bowel Obstruction
A 57-year-old man presents to the emergency department with fever and right upper quadrant pain. On examination, he has a blood pressure of 80/60mmHg, pulse rate of 110 bpm, respiratory rate of 22 breaths per minute and temperature of 38.9°C. The right upper quadrant is tender to palpation. Laboratory studies reveal high white cell count and mildly elevated bilirubin. Based on the findings you suspect acute cholangitis as the diagnosis. Which one of the following organisms is most likely to have caused sepsis?
A. Staphylococcus aureus.
B. Pseudomonas aeruginosa.
C. Enterococcus fecalis.
D. Escherichia coli.
E. Treponema pallidum.
D. Escherichia coli.
Acute cholangitis is a clinical syndrome characterized by fever, abdominal pain and jaundice, developing as a result of stasis and infection in the biliary tract. It can rapidly progress to sepsis. Biliary system obstruction is the most common predisposing factor.
Culture of bile, ductal stones, and blocked biliary stents are positive in over 90% of cases, yielding a mixed growth of gram negative and gram positive bacteria. The most common bacteria isolated have colonic origin.
Escherichia coli is the major bacterium isolated (25 to 50%)
, followed by Klebsiella (15-20% percent) and Enterobacter species (5-10%). Enterococcus species (e.g. enterococcus fecalis) are the most common gram positive bacteria (10-20%).
Anaerobes, such as bacteroides and Clostridia, are usually present as isolated pathogens , but they are rarely the sole infecting organisms and it is not clear if they play a role in acute cholangitis.